// Numbas version: exam_results_page_options {"name": "Find limit of a sequence", "extensions": [], "custom_part_types": [], "resources": [], "navigation": {"allowregen": true, "showfrontpage": false, "preventleave": false, "typeendtoleave": false}, "question_groups": [{"pickingStrategy": "all-ordered", "questions": [{"variable_groups": [{"variables": ["a", "b1", "b", "c", "d"], "name": "x_n"}, {"variables": ["n1", "n2", "n3", "n4"], "name": "Wrong choices for N"}], "variables": {"b": {"templateType": "anything", "group": "x_n", "definition": "if(a*d=b1*c,b1+1,b1)", "description": "", "name": "b"}, "c": {"templateType": "anything", "group": "x_n", "definition": "random(1..20)", "description": "", "name": "c"}, "n1": {"templateType": "anything", "group": "Wrong choices for N", "definition": "N+\nif(\n N>50,\n random(-1,1)*random(8..48#4),\n //otherwise\n random(1,3)\n)", "description": "", "name": "n1"}, "b1": {"templateType": "anything", "group": "x_n", "definition": "random(-1,1)*random(2..9)", "description": "", "name": "b1"}, "a": {"templateType": "anything", "group": "x_n", "definition": "random(2..9)", "description": "", "name": "a"}, "n3": {"templateType": "anything", "group": "Wrong choices for N", "definition": "N+\nif(\n N>50,\n random(-1,1)*random(6..42#4),\n //otherwise\n random(5,6)\n)", "description": "", "name": "n3"}, "n2": {"templateType": "anything", "group": "Wrong choices for N", "definition": "N+\nif(\n N>50,\n random(-1,1)*random(7..43#4),\n //otherwise\n random(2,4)\n)", "description": "", "name": "n2"}, "r": {"templateType": "anything", "group": "Ungrouped variables", "definition": "random(2,3,4)", "description": "

Power of 10 to get within.

", "name": "r"}, "n": {"templateType": "anything", "group": "Ungrouped variables", "definition": "ceil(tval)", "description": "

Smallest $N$ such that $x_n$ is within ep of its limit.

", "name": "n"}, "n4": {"templateType": "anything", "group": "Wrong choices for N", "definition": "N+\nif(\n N>50,\n random(-1,1)*random(5..41#4),\n //otherwise\n random(7,8)\n)", "description": "", "name": "n4"}, "tval": {"templateType": "anything", "group": "Ungrouped variables", "definition": "(1 / c) * ((10 ^ r * abs(b * c -(a * d))) / c -d)", "description": "

Value of $n$ when $x_n$ is exactly $10^{-r}$ away from the limit (not necessarily an integer)

", "name": "tval"}, "d": {"templateType": "anything", "group": "x_n", "definition": "random(1..20)", "description": "", "name": "d"}, "ep": {"templateType": "anything", "group": "Ungrouped variables", "definition": "10^(-r)", "description": "", "name": "ep"}}, "ungrouped_variables": ["r", "ep", "tval", "n"], "question_groups": [{"pickingStrategy": "all-ordered", "questions": [], "name": "", "pickQuestions": 0}], "name": "Find limit of a sequence", "functions": {}, "showQuestionGroupNames": false, "parts": [{"scripts": {}, "gaps": [{"answer": "{a}/{c}", "vsetrange": [0, 1], "checkingaccuracy": 0.001, "showCorrectAnswer": true, "expectedvariablenames": [], "notallowed": {"message": "

Enter your answer as a fraction or integer, not as a decimal.

", "showStrings": false, "partialCredit": 0, "strings": ["."]}, "showpreview": true, "checkingtype": "absdiff", "scripts": {}, "checkvariablenames": false, "type": "jme", "answersimplification": "std", "marks": 1, "vsetrangepoints": 5}], "type": "gapfill", "prompt": "\n \n \n

What is the limit of this sequence?

\n \n \n \n

$\\displaystyle{\\lim_{x\\to\\infty} x_n=\\;\\;}$[[0]]

\n \n \n \n

Input the limit as a fraction or an integer and not a decimal.

\n \n \n", "showCorrectAnswer": true, "marks": 0}, {"scripts": {}, "gaps": [{"displayType": "radiogroup", "choices": ["

{N}

", "

{N1}

", "

{N2}

", "

{N3}

", "

{N4}

"], "displayColumns": 0, "distractors": ["", "", "", "", ""], "shuffleChoices": true, "scripts": {}, "minMarks": 0, "type": "1_n_2", "maxMarks": 0, "showCorrectAnswer": true, "matrix": [1, 0, 0, 0, 0], "marks": 0}], "type": "gapfill", "prompt": "\n \n \n

Which of the following integers has the property that it is the least integer $N$ such that all terms in the sequence are within $10^{\\var{-r}}$ of the limit for all $n \\geq N $?

\n \n \n \n

[[0]]

\n \n \n", "showCorrectAnswer": true, "marks": 0}], "statement": "

Let

\n

\\[ x_n=\\simplify[std]{({a}n+{b})/({c}n+{d})}, \\quad n=1,\\; 2,\\; 3 \\ldots \\]

", "tags": ["checked2015", "convergence of a sequence", "limit", "limit of sequences", "limits", "MAS1601", "sequences", "taking the limit", "tested1"], "rulesets": {"std": ["all", "fractionNumbers", "!collectNumbers", "!noLeadingMinus"]}, "preamble": {"css": "", "js": ""}, "type": "question", "metadata": {"notes": "

4/07/2012:

\n


Changed inequality sign in prompt from $\\lt$ to $\\leq$ and as a consequence changed them in the Advice. Answer remains the same.

\n

21/07/2012:

\n

Added description.

\n

Needs better tags to describe second part.

\n

Also need to redefine the variables so that a and b  and a and c are coprime - results in a better and less clumsy Advice solution. This is the \"changes needed\" tag. Issue raised as having defined a new function chcop using the gcd function, the editor did not register it in the variables list - although the question compiled and ran.

\n

(Contd.) The variables a,b,c,d have been redefined. Also noticed that the MCQ had two correct answers on rare occasions. This has been corrected.

\n

Got rid of the changes needed tag.

\n

27/7/2012:

\n

Added tags.

\n

24/12/2012:

\n

Checked calculation, OK. Added tested1 tag.

", "licence": "Creative Commons Attribution 4.0 International", "description": "

Let $x_n=\\frac{an+b}{cn+d},\\;\\;n=1,\\;2\\ldots$. Find  $\\lim_{x \\to\\infty} x_n=L$ and find least $N$ such that $|x_n-L| \\le 10^{-r},\\;n \\geq N,\\;r \\in \\{2,\\;3,\\;4\\}$.

"}, "variablesTest": {"condition": "gcd(a,b)=1 and gcd(c,d)=1", "maxRuns": 100}, "advice": "

a)

\n

The limit is $\\displaystyle \\simplify[std]{{a}/{c}}$.

\n

b)

\n

To find the least $N$ such that all terms from the the $N$th are within $10^{\\var{-r}}$ of the limit, we proceed as follows:

\n

\\begin{align}
\\left| \\simplify[std]{x_n -({a} / {c})} \\right| \\leq 10^{ -\\var{r}} &\\iff \\left| \\simplify[std]{({a}n+{b})/({c}n+{d})-{a}/{c}} \\right| \\leq 10 ^ { -\\var{r}} \\\\
&\\iff \\simplify[std]{abs({b*c-a*d})/({c^2}n+{c*d})}\\leq 10 ^ { -\\var{r}}
\\end{align}

\n

(We can get rid of the absolute value in the denominator as $\\simplify[std]{{c^2}n+{c*d}} \\gt 0$, $\\forall n=1,\\; 2,\\; 3 \\ldots $)

\n

Rearranging this last inequality by multiplying both sides by $(\\simplify[std]{{c^2}n+{c*d}}) \\times 10^{\\var{r}}$ (this is positive and so the inequality does not reverse), we get:

\n

\\[ \\simplify[std]{{c^2}n+{c*d}} \\geq \\var{10^r*abs(b*c-a*d)} \\iff n \\geq \\simplify[std]{{1}/{c^2}({10^r*abs(b*c-a*d)}-{c*d})}=\\var{tval}\\]

\n

{if(fract(tval)>0,\"The least integer value is given by rounding up, i.e.\",\"So\")} $N=\\var{N}$.

", "contributors": [{"name": "Newcastle University Mathematics and Statistics", "profile_url": "https://numbas.mathcentre.ac.uk/accounts/profile/697/"}]}]}], "contributors": [{"name": "Newcastle University Mathematics and Statistics", "profile_url": "https://numbas.mathcentre.ac.uk/accounts/profile/697/"}]}